LSAT and Law School Admissions Forum

Get expert LSAT preparation and law school admissions advice from PowerScore Test Preparation.

User avatar
 Dave Killoran
PowerScore Staff
  • PowerScore Staff
  • Posts: 5852
  • Joined: Mar 25, 2011
|
#43485
Complete Question Explanation
(The complete setup for this game can be found here: lsat/viewtopic.php?t=7431)

The correct answer choice is (C)

Answer choices (A) and (B) are eliminated by the application of the third rule.

Answer choices (D) and (E) are eliminated because they both contain a pair of inexperienced plumbers.

Answer choice (D) is also eliminated because T must be assigned to a team with G or K.

Answer choice (C) is thus correct.

Get the most out of your LSAT Prep Plus subscription.

Analyze and track your performance with our Testing and Analytics Package.